LSAT and Law School Admissions Forum

Get expert LSAT preparation and law school admissions advice from PowerScore Test Preparation.

 Rachael Wilkenfeld
PowerScore Staff
  • PowerScore Staff
  • Posts: 1358
  • Joined: Dec 15, 2011
|
#71764
Hi t_m6289,

When you think about something that is a necessary assumption. you are looking for what absolutely is required for the argument to work. Our conclusion here is that the practice of predatory pricing SHOULD be acceptable. In order to do that, you need an assumption that shows that specific practice is ok. Your proposed answer that "some predatory pricing practices are acceptable" is very different than saying that THIS practice is acceptable. Saying some practices are ok is not enough to say this one is.

Defending that a particular practice is acceptable requires a fairly strong answer choice. Sometimes people shy away from strong answer choices, but that's not always a great technique. You need to know what sort of question you have, and what that question requires you to do. Answer choice (E) would be a terrible choice if it was asing what must be true---the language is too strong. But when asking what we need in order to draw the conclusion? The language is perfect.

The argument basically holds that since the threat of competition will keep prices from getting unreasonable, predatory pricing is acceptable. Our answer choice (E) states that any pricing practice that does not result in unreasonable prices should be acceptable. Without that answer choice, we wouldn't be able to say that the predatory practices should be acceptable. We need to link the fact that the practices do not result in unreasonable prices with the fact that they should be acceptable. Without strong language linking those two ideas, we cannot support the conclusion.

Hope that helps,
Rachael
 t_m6289
  • Posts: 7
  • Joined: Nov 06, 2019
|
#71767
Thanks for your response Rachael. I'm still a bit confused, maybe because I've be interpreting necessary assumption to mean the bare minimum required for an argument to be true. There are some questions where answer choices go beyond what is required and are incorrect because of that. I want to be sure I understand why this isn't one of those cases.

It's definitely true that saying that some of these practices are acceptable is not the same as saying that this particular practice is acceptable, but it seems to me that the former is all that is necessary for this conclusion to be true. To me it seems like E is a sufficient assumption whereas the "some" language I proposed is what is necessary. For example, if I say:

X is a type of Y.
X is acceptable.

Then wouldn't it be the case that:
NA: some types of Y are acceptable.
SA: any type of Y is acceptable.

I also understand that support assumptions function much like sufficient assumptions (connecting a premise to a conclusion). Am I misunderstanding something about how supporters work?

Thanks for your help!
 Rachael Wilkenfeld
PowerScore Staff
  • PowerScore Staff
  • Posts: 1358
  • Joined: Dec 15, 2011
|
#71824
Hi t_m6289,

To me, it sounds like you are confusing must be true question and assumption questions. They are fundamentally different and asking you to look at different things. For must be true questions, you are looking for an answer that must be true based on the stimulus. For assumption questions, you are looking for an answer that is required in order for the conclusion to be true. The flow of information is different. In must be true questions, you are moving from the stimulus to the answer choices. That flips for assumption questions, and you are using the answer choices to help the stimulus.

Let's look at a simple argument to see how this would play out.
Premise: Socrates is a human.
Conclusion: Socrates is mortal.

If I am looking for a must be true answer, I can't go beyond what I've been told in the stimulus. We can't add information, so our answer choice is going to be very close to the facts we have in this example. Something like "Socrates is able to die" would be a good answer because it's just using the definition of a term in the argument. We couldn't say much else, because the facts, as given, don't connect particularly well.

If we are looking for an assumption question, on the other hand, we are going to need to connect those facts. Assumptions by their nature add to the argument. What would be a great assumption here? Something like "All humans are mortal" would be required to link the premise and the conclusion. It's necessary for the conclusion to follow. Some humans are mortal wouldn't be enough to conclude that Socrates is. We need that strong language in the assumption.

It's true that assumptions are asking for what's required. There's not really a ranking though---either something is required or it isn't, so there's no "bare minimum" to assumptions. A statement either is or is not necessary.

In this question, our structure is sort of like this.

Premise: Some companies engage in predatory pricing.
Premise: Threat of competition will stop those companies from raising prices to unreasonable levels
Conclusion predatory pricing should be acceptable

That's different than your example because we don't have any types. We have pricing won't be unreasonable, therefore it should be acceptable. We have a multiple of assumptions that would be required here---but the only one of these listed as an answer choice.

1) Pricing practices can be acceptable
2) If it's not unreasonable pricing practice, then it is acceptable

Both of the above are needed. It's not just that some are acceptable, it's that this specific one is acceptable. Thinking of it conditionally may make it more clear for you.

Hope that helps!
Rachael
User avatar
 WildMountainElk
  • Posts: 6
  • Joined: Feb 01, 2022
|
#94893
I've thought a lot about this question, and I still haven't been able to wrap my head around why answer choice E is necessary for the argument to be valid. I interpret the word "any" in answer choice E to be equivalent to "all" (maybe this is my mistake?).

The way I see it, there may be other pricing practices that do no result in unreasonable prices, but the argument in the question does not require that all such practices be acceptable. In fact, they could all be unacceptable for various reasons, and that would not invalidate the argument, because predatory pricing could still be the lone exception.

I 100% agree that answer choice E is sufficient for the argument to be valid, and it is the only such answer choice. Does the fact that it's the only sufficient answer choice indicate that it must be the correct one?
User avatar
 WildMountainElk
  • Posts: 6
  • Joined: Feb 01, 2022
|
#94894
Sorry for the consecutive post, but I think I've wrapped my head around this now.

If we peal away all of the "fluff" around the conditional statement in the stem, we're left with this:

if not unreasonable prices then acceptable pricing practice
or if we remove the double negative:
if reasonable prices then acceptable pricing practice

That's it, that's what the argument depends on. So, if there were even a single instance where a pricing practice resulted in reasonable prices but was deemed unacceptable, that conditional statement no longer holds. Consequently, the argument does indeed depend on the assumption that any/all pricing practices that result in reasonable prices are acceptable.

Is my logic sound here?
 Robert Carroll
PowerScore Staff
  • PowerScore Staff
  • Posts: 1787
  • Joined: Dec 06, 2013
|
#94914
WildMountainElk,

Your second post is exactly correct. Good job on untangling this difficult situation! Even after a lot of thought, answer choice (E) does look too strong to be a Necessary Assumption for the argument. However, it is - the author thought that all they had to do to prove their conclusion was to show that unreasonable prices would not result from a particular practice. The author thought "That's good enough; as long as I've shown unreasonable prices won't result, there is nothing more my argument needs to do." So the author apparently is committed to the idea that a lack of unreasonable prices alone makes a pricing practice acceptable - that's why the strong language of answer choice (E) is actually necessary!

Well done again - this one was tough!

Robert Carroll
User avatar
 Cantthinkofagood1
  • Posts: 6
  • Joined: Sep 04, 2023
|
#103019
I am still very confused about this question. Below is my train of thought, please tell me where I made my error in reasoning.

Asking for "on what assumption that the argument depends" is basically asking for what must be true if the argument is correct. The argument in this question seems to be something like "If the threat of renewed competition can keep prices from becoming unreasonable, then the predatory pricing should be acceptable". Let's assume E is correct, which I still don't see why, then the negation of E is "NOT any pricing practice that does not result in unreasonable prices should be acceptable". According to the PowerScore LR book "any" is in the category of "all", so it translates to "not all pricing practice that does not result in unreasonable prices should be acceptable" which becomes "some pricing practice that does not result in unreasonable practices should not be acceptable", because "not all does X" is the same as "some don't do X". To me at least, this in no way negates the argument, because "some pricing practice that doesn't result in unreasonable prices should not be acceptable" has nothing to do with whether predatory pricing, which also doesn't result in unreasonable prices, should be acceptable or not. For example, "some A shouldn't do B" does not negate or prevent any particular A from doing B. In this case just because some pricing practices shouldn't be acceptable doesn't mean a particular pricing practice (in this case predatory pricing practice) shouldn't be acceptable. So E fails the negation test.

Sorry for the wall of text, but am I missing something crucial here? When I did this question, I just guessed, because no answer looked good.
User avatar
 Cantthinkofagood1
  • Posts: 6
  • Joined: Sep 04, 2023
|
#103021
I think I realized why E is a good answer after sleeping on it. Let me know if my logic is right.

I originally thought, like many people had mentioned in this thread, that E seemed too strong of an answer at first. For example, had E said "at least some pricing practice that don't result in unreasonable prices should be acceptable, E would have been the perfect answer. But the trick to this problem is actually realizing that the range choices for pricing practices isn't that extensive. It is honestly just too high, within competitive range, or too low. Too high is just economic suicide, so no business would try it. Within competitive range isn't an issue and no sane person would argue that it isn't acceptable. The only pricing practice that is even in consideration for whether it could be acceptable or not is "too low" which this problem defines as predatory. So, the set of all possibility that can be included in "any" for this problem is a set of one.

A simpler analogy that proves my point is about a jar of coins. The LSAT argument would be "if the number of coins in the jar were even which means it's divisible by 2, then the child will be happy". The argument depends on the assumption "Any parity that makes the number of coins in the jar divisible by 2 will make the child happy" Despite the word "any" seeming strong, it really only has two possibilities, odd or even. Since odd can't be divisible by 2, the only option leaves "even", so the assumption is just a repeat of the argument. Thus, it must be true if the argument were true.

I think this level of insight is too much for a mere mortal like me to have in the time limit of the LSAT. If I see a question of this level of difficulty on the real LSAT, I will just have to accept the fact that I can't get it right on time.
User avatar
 Jonathan Evans
PowerScore Staff
  • PowerScore Staff
  • Posts: 726
  • Joined: Jun 09, 2016
|
#103052
Hi, cantthinkofagood1!

On the contrary, I think you're fully capable of getting this right within the time limit!

The argument involves the conditional assumption that:

No threat of unreasonable prices :arrow: predatory pricing acceptable

This is more or less a restatement of answer choice (E).

You're right that this strong proposition implies the truth of the proposition that there exists a non-unreasonable pricing practice that should be acceptable, but if I may make my own ethical judgment, we should expect the LSAT to go more maximalist here. Why stop with some when all is right there?

This is a challenging question but with appropriate prephrasing and analysis, this is 100% something you can handle.

Get the most out of your LSAT Prep Plus subscription.

Analyze and track your performance with our Testing and Analytics Package.